Вы находитесь на странице: 1из 63

CHAPTER 21

CAPITAL BUDGETING AND COST ANALYSIS

21-1 “Capital budgeting has the same focus as accrual accounting.” Do you agree? Explain.

No. Capital budgeting focuses on an individual investment project throughout its life,
recognizing the time value of money. The life of a project is often longer than a year. Accrual
accounting focuses on a particular accounting period, often a year, with an emphasis on income
determination.

21-2 List and briefly describe each of the five stages in capital budgeting.

The five stages in capital budgeting are the following:


1. An identification stage to determine which types of capital investments are available to
accomplish organization objectives and strategies.
2. An information-acquisition stage to gather data from all parts of the value chain in order to
evaluate alternative capital investments.
3. A forecasting stage to project the future cash flows attributable to the various capital
projects.
4. An evaluation stage where capital budgeting methods are used to choose the best
alternative for the firm.
5. A financing, implementation and control stage to fund projects, get them under way and
monitor their performance.

21-3 What is the essence of the discounted cash flow methods?

In essence, the discounted cash-flow method calculates the expected cash inflows and outflows
of a project as if they occurred at a single point in time so that they can be aggregated (added,
subtracted, etc.) in an appropriate way. This enables comparison with cash flows from other
projects that might occur over different time periods.
21-4 “Only quantitative outcomes are relevant in capital budgeting analyses.” Do you
agree? Explain.

No. Only quantitative outcomes are formally analyzed in capital budgeting decisions. Many
effects of capital budgeting decisions, however, are difficult to quantify in financial terms.
These nonfinancial or qualitative factors (for example, the number of accidents in a
manufacturing plant or employee morale) are important to consider in making capital budgeting
decisions.
21-5 How can sensitivity analysis be incorporated in DCF analysis?

Sensitivity analysis can be incorporated into DCF analysis by examining how the DCF of each
project changes with changes in the inputs used. These could include changes in revenue
assumptions, cost assumptions, tax rate assumptions, and discount rates.
21-6 What is the payback method? What are its main strengths and weaknesses?

21-1
The payback method measures the time it will take to recoup, in the form of expected future net
cash inflows, the net initial investment in a project. The payback method is simple and easy to
understand. It is a handy method when screening many proposals and particularly when
predicted cash flows in later years are highly uncertain. The main weaknesses of the payback
method are its neglect of the time value of money and of the cash flows after the payback period.
The first drawback, but not the second, can be addressed by using the discounted payback
method.
21-7 Describe the accrual accounting rate-of-return method. What are its main strengths and
weaknesses?

The accrual accounting rate-of-return (AARR) method divides an accrual accounting measure of
average annual income of a project by an accrual accounting measure of investment. The
strengths of the accrual accounting rate of return method are that it is simple, easy to understand,
and considers profitability. Its weaknesses are that it ignores the time value of money and does
not consider the cash flows for a project.

21-8 “The trouble with discounted cash flow methods is that they ignore depreciation.” Do you
agree? Explain.

No. The discounted cash-flow techniques implicitly consider depreciation in rate of return
computations; the compound interest tables automatically allow for recovery of investment. The
net initial investment of an asset is usually regarded as a lump-sum outflow at time zero. Where
taxes are included in the DCF analysis, depreciation costs are included in the computation of the
taxable income number that is used to compute the tax payment cash flow.
21-9 “Let’s be more practical. DCF is not the gospel. Managers should not become so
enchanted with DCF that strategic considerations are overlooked.” Do you agree? Explain.

A point of agreement is that an exclusive attachment to the mechanisms of any single method
examining only quantitative data is likely to result in overlooking important aspects of a
decision.
Two points of disagreement are (1) DCF can incorporate those strategic considerations that
can be expressed in financial terms, and (2) “Practical considerations of strategy” not expressed
in financial terms can be incorporated into decisions after DCF analysis.

21-10 “All overhead costs are relevant in NPV analysis.” Do you agree? Explain.

All overhead costs are not relevant in NPV analysis. Overhead costs are relevant only if the
capital investment results in a change in total overhead cash flows. Overhead costs are not
relevant if total overhead cash flows remain the same but the overhead allocated to the particular
capital investment changes.
21-11 Bill Watts, president of Western Publications, accepts a capital budgeting project
proposed by division X. This is the division in which the president spent his first 10 years with
the company. On the same day, the president rejects a capital budgeting project proposal from
division Y. The manager of division Y is incensed. She believes that the division Y project has
an internal rate of return at least 10 percentage points higher than the division X project. She

21-2
comments, “What is the point of all our detailed DCF analysis? If Watts is panting over a
project, he can arrange to have the proponents of that project massage the numbers so that it
looks like a winner.” What advice would you give the manager of division Y?

The Division Y manager should consider why the Division X project was accepted and the
Division Y project rejected by the president. Possible explanations are:
a. The president considers qualitative factors not incorporated into the IRR computation
and this leads to the acceptance of the X project and rejection of the Y project.
b. The president believes that Division Y has a history of overstating cash inflows and
understating cash outflows.
c. The president has a preference for the manager of Division X over the manager of
Division Y—this is a corporate politics issue.
Factor a. means qualitative factors should be emphasized more in proposals. Factor b. means
Division Y needs to document whether its past projections have been relatively accurate. Factor
c. means the manager of Division Y has to play the corporate politics game better.

21-12 Distinguish different categories of cash flows to be considered in an equipment-


replacement decision by a taxpaying company.

The categories of cash flow that should be considered in an equipment-replacement decision are:
1a. Initial machine investment,
b. Initial working-capital investment,
c. After-tax cash flow from current disposal of old machine,
2a. Annual after-tax cash flow from operations (excluding the depreciation effect),
b. Income tax cash savings from annual depreciation deductions,
3a. After-tax cash flow from terminal disposal of machines, and
b. After-tax cash flow from terminal recovery of working-capital investment.

21-13 Describe three ways income taxes can affect the cash inflows or outflows in a motor-
vehicle-replacement decision by a taxpaying company.

Income taxes can affect the cash inflows or outflows in a motor vehicle replacement decision as
follows:
a. Tax is payable on gain or loss on disposal of the existing motor vehicle,
b. Tax is payable on any change in the operating costs of the new vehicle vis-à-vis the
existing vehicle, and
c. Tax is payable on gain or loss on the sale of the new vehicle at the project termination
date.
d. Additional depreciation deductions for the new vehicle result in tax cash savings.

21-14 How can capital budgeting tools assist in evaluating a manager who is responsible for
retaining customers of a cellular telephone company?

A cellular telephone company manager responsible for retaining customers needs to consider the
expected future revenues and the expected future costs of “different investments” to retain

21-3
customers. One such investment could be a special price discount. An alternative investment is
offering loyalty club benefits to long-time customers.

21-15 Distinguish the nominal rate of return from the real rate of return.

These two rates of return differ in their elements:

Real-rate of return Nominal rate of return


1. Risk-free element 1. Risk-free element
2. Business-risk element 2. Business-risk element
3. Inflation element

The inflation element is the premium above the real rate of return that is demanded for the
anticipated decline in the general purchasing power of the monetary unit.

21-16 A company should accept for investment all positive NPV investment alternatives when
which of the following conditions is true?

a. The company has extremely limited resources for capital investment.


b. The company has excess cash on its balance sheet.
c. The company has virtually unlimited resources for capital investment.
d. The company has limited resources for capital investment but is planning to issue new equity
to finance additional capital investment.

SOLUTION

Choice "c" is correct. A company with unlimited resources for capital investments should accept
all positive NPV investment options. Each positive NPV investment will increase shareholder
value.

Choice "a" is incorrect. Any company with limited resources for capital investment must ration
scarce capital investment funding. Investment options must be carefully prioritized.

Choice "b" is incorrect. Excess cash does not mean the company has unlimited funds for capital
investment.

Choice "d" is incorrect. A company planning to issue new equity may have additional funding
following the equity issue but presently must ration scarce capital investment funds.

21-17 Which of the following items describes a weakness of the internal rate-of-return method?

a. The internal rate of return is difficult to calculate and requires a financial calculator or
spreadsheet tool such as Excel to calculate efficiently.
b. Cash flows from the investment are assumed in the IRR analysis to be reinvested at the
internal rate of return.
c. The internal rate-of-return calculation ignores time value of money.

21-4
d. The internal rate-of-return calculation ignores project cash flows occurring after the initial
investment is recovered.

SOLUTION

Choice "b" is correct. Assuming funds can be reinvested at the computed internal rate of return
may not be a realistic assumption if the internal rate of return is unrealistically high or
unrealistically low.

Choice "a" is incorrect. While the IRR is somewhat difficult to calculate, this is not a weakness
of the IRR computation.

Choice "c" is incorrect. The IRR calculation considers time value of money.

Choice "d" is incorrect. The IRR calculation considers all estimated project cash flows.

21-18 Which of the following statements is true if the NPV of a project is –$4,000 (negative
$4,000) and the required rate of return is 5 percent?

a. The project’s IRR is less than 5 percent.


b. The required rate of return is lower than the IRR.
c. The NPV assumes cash flows are reinvested at the IRR.
d. The NPV would be positive if the IRR was equal to 5 percent.

SOLUTION

Choice "a" is correct. When the net present value (NPV) of a project is negative, that necessarily
implies that the internal rate of return (IRR) is less than the required rate of return. Here, since
the required rate of return is 5 percent, it must be the case that the IRR is less than 5 percent.

Choice "b" is incorrect. If the required rate of return was lower than the IRR, the NPV would be
positive.

Choice "c" is incorrect. The NPV assumes that cash flows are reinvested at the required rate of
return, rather than at the internal rate of return.

Choice "d" is incorrect. The NPV would be equal to zero if the IRR was equal to the required
rate of return (5 percent in this question).

21-19 The following information pertains to the January 2, year 2 transaction replacing a print
machine for Hidden Creek Enterprises, Inc.

Net book value – old print machine $20,000


Total cost of new machine $180,000
Down payment on new machine $35,000
Sale price of old machine $30,000

21-5
Tax rate 30%

What is the net total of relevant costs on January 2, year 2?

a. $173,000 b. $153,000
c. $28,000 d. 8,000.

SOLUTION

Choice "b" is correct. The relevant costs are as follows:

Acquisition cost of new machine $180,000


Sale of old machine – cash received (30,000)
Tax on sale of old machine 3,000
Relevant cost $153,000

The total acquisition price of the asset is considered to be a relevant cost. The financing method
and cost of financing are analyzed separately.

The cash received on the sale of the old asset is the sales price of the machine, less the tax paid
on the gain. The gain on sale is the sales price less the net book value of the old asset.

Choice "a" is incorrect. The cash received on the old asset when it is sold is a component of the
relevant cost. The gain on the asset sale is only important to calculate the amount of tax due on
the sale transaction.

Choice "c" is incorrect. The total cost of the new asset, not just the down payment made at the
time of purchase, is considered to be a relevant cost in the replacement decision. The cash
received on the old asset when it is sold is a relevant cost. The gain on the asset sale is only
important to calculate the amount of tax due on the sale transaction.

Choice "d" is incorrect. The total cost of the new asset, not just the down payment made at the
time of purchase, is considered to be a relevant cost in the replacement decision.

21-20 Nick’s Enterprises has purchased a new machine tool that will allow the company to
improve the efficiency of its operations. On an annual basis, the machine will produce 20,000
units with an expected selling price of $10, prime costs of $6 per unit, and a fixed cost allocation
of $3 per unit. Annual depreciation on the machine is $12,000, and the tax rate of the company is
25%.

What is the annual cash flow generated from the new machine?

a. $63,000 b. $51,000
c. $18,000 d. $6,000

21-6
SOLUTION

Choice "a" is correct. The cash flow generated is calculated as follows:

Sales generated ($10 × 20,000 units) $200,000


Prime costs ($6 × 20,000) (120,000)
Cash generated from sales 80,000
Tax increase on cash generated ($80,000 × 25%) (20,000)
Tax decrease as a result of depreciation ($12,000 × 25%) 3,000
Cash flow generated $ 63,000

The allocation of fixed costs is not a relevant cost since the costs would have been incurred
whether or not the company has the new machine tool.

Choice "b" is incorrect. Depreciation is not subtracted from the amount of cash generated since it
is a noncash expense. Only the tax benefit of the depreciation deduction is considered as an
addition to the cash flow generated.

Choice "c" is incorrect. Fixed costs exist independently of the new machine and are an
accounting allocation. The fixed costs are not considered in the cash flow calculation for the
machine tool.

Choice "d" is incorrect. Fixed costs which exist independently of the new machine and are an
accounting allocation are not considered in the cash flow calculation for the machine tool.
Depreciation is not subtracted from the amount of cash generated since it is a noncash expense.
Only the tax benefit of the depreciation deduction is considered as an addition to the cash flow
generated.

21-21 Exercises in compound interest, no income taxes. To be sure that you understand
how to use the tables in Appendix A at the end of this book, solve the following exercises. Ignore
income tax considerations. The correct answers, rounded to the nearest dollar, appear on page
875.

Required:
1. You have just won $50,000. How much money will you accumulate at the end of 5 years if
you invest it at 6% compounded annually? At 12%?
2. Twelve years from now, the unpaid principal of the mortgage on your house will be $249,600.
How much do you need to invest today at 6% interest compounded annually to accumulate the
$249,600 in 12 years?
3. If the unpaid mortgage on your house in 12 years will be $249,600, how much money do you
need to invest at the end of each year at 6% to accumulate exactly this amount at the end of
the 12th year?
4. You plan to save $4,800 of your earnings at the end of each year for the next 8 years. How
much money will you accumulate at the end of the 8th year if you invest your savings
compounded at 4% per year?
5. You have just turned 65 and an endowment insurance policy has paid you a lump sum of
$400,000. If you invest the sum at 6%, how much money can you withdraw from your

21-7
account in equal amounts at the end of each year so that at the end of 7 years (age 72), there
will be nothing left?
6. You have estimated that for the first 6 years after you retire you will need a cash inflow of
$48,000 at the end of each year. How much money do you need to invest at 4% at your
retirement age to obtain this annual cash inflow? At 6%?
7. The following table shows two schedules of prospective operating cash inflows, each of
which requires the same net initial investment of $18,000 now:

Annual Cash Inflows


Year Plan A Plan B
1 $ 2,000 $ 3,000
2 3,000 5,000
3 4,000 9,000
4 7,000 5,000
5 9,000 3,000
Total $25,000 $25,000

The required rate of return is 6% compounded annually. All cash inflows occur at the end of
each year. In terms of net present value, which plan is more desirable? Show your computations.

SOLUTION

Exercises in compound interest, no income taxes.


The answers to these exercises are printed after the last problem, at the end of the chapter.

21-22 Capital budgeting methods, no income taxes. Yummy Candy Company is considering
purchasing a second chocolate dipping machine in order to expand their business. The
information Yummy has accumulated regarding the new machine is:

Cost of the machine $80,000


Increased annual contribution margin $15,000
Life of the machine 10 years
Required rate of return 6%

Yummy estimates they will be able to produce more candy using the second machine and thus
increase their annual contribution margin. They also estimate there will be a small disposal value
of the machine but the cost of removal will offset that value. Ignore income tax issues in your
answers. Assume all cash flows occur at year-end except for initial investment amounts.

Required:
1. Calculate the following for the new machine:

21-8
a.
Net present value
b.
Payback period
c.
Discounted payback period
d.
Internal rate of return (using the interpolation method)
e.
Accrual accounting rate of return based on the net initial investment (assume straight-line
depreciation)
2. What other factors should Yummy Candy consider in deciding whether to purchase the new
machine?

SOLUTION

(20–25 min.) Capital budgeting methods, no income taxes.

1a. The table for the present value of annuities (Appendix A, Table 4) shows:
$1 for 10 periods at 6% = 7.36

Net present value = $15,000 (7.36) – $80,000

= $110,400 – $80,000 = $30,400

1b. Payback period = $80,000 ÷ $15,000 = 5.33 years

1c. Discounted Payback Period

Period Cash Savings Discount Discounted Cumulative Unrecovered


Factor (6%) Cash Savings Discounted Investment
Cash Savings
0 -$80,000
1 $15,000 0.943 $14,145 $14,145 ($65,855)
2 $15,000 0.890 $13,350 $27,495 ($52,505)
3 $15,000 0.840 $12,600 $40,095 ($39,905)
4 $15,000 0.792 $11,880 $51,975 ($28,025)
5 $15,000 0.747 $11,205 $63,180 ($16,820)
6 $15,000 0.705 $10,575 $73,755 ($ 6,245)
7 $15,000 0.665 $ 9,975 $83,730

$6,245/$9,975 = .6261
Discounted Payback period = 6.63 years

1d. Internal rate of return:


$80,000 = Present value of annuity of $15,000 at R% for 10 years, or
what factor (F) in the table of present values of an annuity
(Appendix A, Table 4) will satisfy the following equation.

$80,000 = $15,000F

21-9
F = 80,000/15,000= 5.333

On the 10-year line in the table for the present value of annuities (Appendix A, Table 4), find the
column closest to 5.333; it is between a rate of return of 12% and 14%.

Interpolation is necessary:
Present Value Factors
12% 5.650 5.650
IRR rate – 5.333
14% 5.216 ––
Difference 0.434 0.317

Internal rate of return = 12% + (0.317/0.434) * (2%)

= 12% + .730 (2%) = 13.46%

1d. Accrual accounting rate of return based on net initial investment:


Net initial investment = $80,000
Estimated useful life = 10 years
Annual straight-line depreciation = $80,000 ÷ 10 = $8,000
Accrual accounting rate of return =
Increase in expected average annual operating income
Net initial investment

= ($15,000 – $8,000) / $80,000 = $7,000 / $80,000 = 8.75%

Note how the accrual accounting rate of return can produce results that differ markedly from the
internal rate of return.

2. Other than the NPV, rate of return and the payback period on the new computer system,
factors that Yummy should consider are:

 Issues related to the financing the project, and the availability of capital to pay for the
system.
 The benefits of the machine for customers (more customers, more variety of products,
faster production time, special orders).
 Ensure that the costs of training and other “hidden” start-up costs are included in the
estimated $80,000 cost of the new machine.
 Verify that the technology of the machine will last for the estimated 10 years.

21-23 Capital budgeting methods, no income taxes. City Hospital, a nonprofit organization,
estimates that it can save $28,000 a year in cash operating costs for the next 10 years if it buys a
special-purpose eye-testing machine at a cost of $110,000. No terminal disposal value is

21-10
expected. City Hospital’s required rate of return is 14%. Assume all cash flows occur at year-end
except for initial investment amounts. City Hospital uses straight-line depreciation.

Required:
1. Calculate the following for the special-purpose eye-testing machine:
a. Net present value
b. Payback period
c. Internal rate of return
d. Accrual accounting rate of return based on net initial investment
e. Accrual accounting rate of return based on average investment
2. What other factors should City Hospital consider in deciding whether to purchase the special-
purpose eye-testing machine?

SOLUTION

(25 min.) Capital budgeting methods, no income taxes.

The table for the present value of annuities (Appendix A, Table 4) shows:
10 periods at 14% = 5.216

1a. Net present value = $28,000 (5.216) – $110,000


= $146,048 – $110,000 = $36,048

$110,000
b. Payback period = $28 ,000 = 3.93 years

c. For a $110,000 initial outflow, the project generates $28,000 in cash flows at the end of
each of years one through ten.
Using either a calculator or Excel, the internal rate of return for this stream of cash flows is
found to be 21.96%.

d. Accrual accounting rate of return based on net initial investment:


Net initial investment = $110,000
Estimated useful life = 10 years
Annual straight-line depreciation = $110,000 ÷ 10 = $11,000
$28,000 − $ 11,000
Accrual accounting rate of return = $110 ,000
$17 ,000
= $110,000 = 15.45%

e. Accrual accounting rate of return based on average investment:


Average investment = ($110,000 + $0) / 2
= $55,000

21-11
$28,000  $11,000
Accrual accounting rate of return = $55,000 = 30.91%.

2. Factors City Hospital should consider include:


a. Quantitative financial aspects.
b. Qualitative factors, such as the benefits to its customers of a better eye-testing machine
and the employee-morale advantages of having up-to-date equipment.
c. Financing factors, such as the availability of cash to purchase the new equipment.

21-24 Capital budgeting, income taxes. Assume the same facts as in Exercise 21-23 except
that City Hospital is a taxpaying entity. The income tax rate is 30% for all transactions that affect
income taxes.

Required:
1. Do requirement 1 of Exercise 21-23.
2. How would your computations in requirement 1 be affected if the special-purpose machine
had a $10,000 terminal disposal value at the end of 10 years? Assume depreciation
deductions are based on the $110,000 purchase cost and zero terminal disposal value using
the straight-line method. Answer briefly in words without further calculations.

SOLUTION

(35 min.) Capital budgeting, income taxes.

1a. Net after-tax initial investment = $110,000

Annual after-tax cash flow from operations (excluding the depreciation effect):

Annual cash flow from operation with new machine $28,000


Deduct income tax payments (30% of $28,000) 8,400
Annual after-tax cash flow from operations $19,600

Income tax cash savings from annual depreciation deductions


30%  $11,000 $3,300

These three amounts can be combined to determine the NPV:

Net initial investment;


$110,000  1.00 $(110,000)
10-year annuity of annual after-tax cash flows from operations;
$19,600  5.216 102,234
10-year annuity of income tax cash savings from annual depreciation deductions;
$3,300  5.216 17,213
Net present value $ 9,447

21-12
b. Payback period

$110, 000
= ($19,600 + $3,300)

$110,000
= $22 ,900

= 4.80 years

c. For a $110,000 initial outflow, the project now generates $22,900 in after-tax cash flows at
the end of each of years one through ten.

Using either a calculator or Excel, the internal rate of return for this stream of cash flows is
found to be 16.17%.

d. Accrual accounting rate of return based on net initial investment:

$22,900 − $11 ,000 $11,900


AARR = $110 ,000 = $110,000

= 10.82%

e. Accrual accounting rate of return based on average investment:

$22,900  $11, 000 $11,900


AARR = $55, 000 = $55, 000

= 21.64%

2a. Increase in NPV.


To get a sense for the magnitude, note that from Table 2, the present value factor for 10 periods
at 14% is 0.270. Therefore, the $10,000 terminal disposal price at the end of 10 years would
have an after-tax NPV of:

$10,000  (1  0.30)  0.270 = $1,890

b. No change in the payback period of 4.80 years. The cash inflow occurs at the end of year
10.

21-13
c. Increase in internal rate of return. The $10,000 terminal disposal price would raise the
IRR because of the additional inflow. (The new IRR is 16.54%.)

d. The AARR on net initial investment would increase because accrual accounting income
in year 10 would increase by the $7,000 ($10,000 gain from disposal, less 30%  $10,000) after-
tax gain on disposal of equipment. This increase in year 10 income would result in higher
average annual accounting income in the numerator of the AARR formula.

e. The AARR on average investment would also increase, for the same reasons given in the
previous answer. Note that the denominator is unaffected because the investment is still
depreciated down to zero terminal disposal value, and so the average investment remains
$55,000.

21-25 Capital budgeting with uneven cash flows, no income taxes. America Cola is
considering the purchase of a special-purpose bottling machine for $65,000. It is expected to
have a useful life of 4 years with no terminal disposal value. The plant manager estimates the
following savings in cash operating costs:

Year Amount
1 $25,000
2 22,000
3 21,000
4 20,000
Total $88,000

America Cola uses a required rate of return of 18% in its capital budgeting decisions. Ignore
income taxes in your analysis. Assume all cash flows occur at year-end except for initial
investment amounts.

Required:
Calculate the following for the special-purpose bottling machine:

1. Net present value


2. Payback period
3. Discounted payback period
4. Internal rate of return (using the interpolation method)
5. Accrual accounting rate of return based on net initial investment (Assume straight-line
depreciation. Use the average annual savings in cash operating costs when computing the
numerator of the accrual accounting rate of return.)

SOLUTION

(25 min.) Capital budgeting with uneven cash flows, no income taxes.

21-14
1. Present value of savings in cash operating costs:
$25,000 × 0.847 $ 21,175
22,000 × 0.718 5,796
21,000 × 0.609 12,789
20,000 × 0.516 10,320
Present value of savings in cash operating costs 60,080
Net initial investment (65,000)
Net present value $ (4,920)

2. Payback period:
Cumulative Initial Investment Yet to Be
Year Cash Savings Cash Savings Recovered at End of Year
0 – – $65,000
1 $25,000 $25,000 40,000
2 22,000 47,000 18,000
3 21,000 68,000 -

$18,000
Payback period = 2 years + $21,000 = 2.86 years

3. Discounted Payback Period

Period Cash Disc Factor Discounted Cumulative Unrecovered


Savings (18%) Cash Savings Discounted. Investment
Cash Savings
0 -$65,000
1 $25,000 .847 $21,175 $21,175 -$43,825
2 $22,000 .718 $15,796 $36,971 -$28,029
3 $21,000 .609 $12,789 $49,760 -$15,240
4 $20,000 .516 $10,320 $60,080 -$ 4,920

At a 18% rate of return, this project does not save enough to make it worthwhile using the
discounted payback method.

4. From requirement 1, the net present value is negative with a 18% required rate of return.
Therefore, the internal rate of return must be less than 18%.
P.V. P.V. P.V.
Cash P.V. Factor at 14% P.V. Factor at 12% P.V. Factor at 10%
Year Savings at 14% (4) = at 12% (6) = at 10% (8) =
(1) (2) (3) (2) × (3) (5) (2) × (5) (7) (2) × (7)
1 $25,000 0.877 $ 21,925 0.893 $22,325 0.909 $22,725
2 22,000 0.769 16,918 0.797 17,534 0.826 18,172
3 21,000 0.675 14,175 0.712 14,952 0.751 15,771
4 20,000 0.592 11,840 0.636 12,720 0.683 13,660
$64,858 $67,531 $70,328

21-15
Net present value at 14% = $64,858 – $65,000 = $(142)
Net present value at 12% = $67,531 – $65,000 = $2,531
 2,531 
 2,531  142 
Internal rate of return = 12% +   × (2%)

= 12% + (0.947) × (2%) = 13.89%

5. Accrual accounting rate of return based on net initial investment:

$88,000
Average annual savings in cash operating costs = 4 years = $22,000

$65,000
Annual straight-line depreciation = 4 years = $16,250

$22,000−$16,250
Accrual accounting rate of return = $65,000

$5,750
= $65,000 = 8.85%

21-26 Comparison of projects, no income taxes. (CMA, adapted) New Pharm Corporation is
a rapidly growing biotech company that has a required rate of return of 14%. It plans to build a
new facility in Santa Clara County. The building will take 2 years to complete. The building
contractor offered New Pharm a choice of three payment plans, as follows:

■ Plan I: Payment of $175,000 at the time of signing the contract and $4,700,000 upon
completion of the building. The end of the second year is the completion date.
■ Plan II: Payment of $1,625,000 at the time of signing the contract and $1,625,000 at the end
of each of the two succeeding years.
■ Plan III: Payment of $325,000 at the time of signing the contract and $1,500,000 at the end
of each of the three succeeding years.

Required:
1. Using the net present value method, calculate the comparative cost of each of the three
payment plans being considered by New Pharm.
2. Which payment plan should New Pharm choose? Explain.
3. Discuss the financial factors, other than the cost of the plan, and the nonfinancial factors that
should be considered in selecting an appropriate payment plan.

21-16
SOLUTION

(30 min.) Comparison of projects, no income taxes.

1.
Total Present Value Year
Present Discount
Value Factors at 14% 0 1 2 3
Plan I
$ (175,000) 1.000 $ (175,000)
(3,614,300) 0.769 $(4,700,000)
$(3,789,300)

Plan II
$(1,625,000) 1.000 $(1,625,000)
(1,425,125) 0.877 $(1,625,000)
(1,249,625) 0.769 $(1,625,000)
$(4,299,750)

Plan III
$ (325,000) 1.000 $ (325,000)
(1,315,500) 0.877 $(1,500,000)
(1,153,500) 0.769 $(1,500,000)
(1,012,500) 0.675 $(1,500,000)
$(3,806,500)

2. Plan I has the lowest net present value cost, and is therefore preferable on financial
criteria.

3. Factors to consider, in addition to NPV, are:

a. Financial factors including:


 Competing demands for cash.
 Availability of financing for project.

b. Nonfinancial factors including:


 Risk of building contractor not remaining solvent. Plan II exposes New Pharm
most if the contractor becomes bankrupt before completion because it requires
more of the cash to be paid earlier.
 Ability to have leverage over the contractor if quality problems arise or delays in
construction occur. Plans I and III give New Pharm more negotiation strength by
being able to withhold sizable payment amounts if, say, quality problems arise in
Year 1.
 Investment alternatives available. If New Pharm has capital constraints, the new
building project will have to compete with other projects for the limited capital
available.

21-17
21-27 Payback and NPV methods, no income taxes. (CMA, adapted) Andrews Construction
is analyzing its capital expenditure proposals for the purchase of equipment in the coming year.
The capital budget is limited to $5,000,000 for the year. Lori Bart, staff analyst at Andrews, is
preparing an analysis of the three projects under consideration by Corey Andrews, the
company’s owner.

Required:
1. Because the company’s cash is limited, Andrews thinks the payback method should be used
to choose between the capital budgeting projects.
a. What are the benefits and limitations of using the payback method to choose between
projects?
b. Calculate the payback period for each of the three projects. Ignore income taxes. Using
the payback method, which projects should Andrews choose?
2. Bart thinks that projects should be selected based on their NPVs. Assume all cash flows
occur at the end of the year except for initial investment amounts. Calculate the NPV for
each project. Ignore income taxes.
3. Which projects, if any, would you recommend funding? Briefly explain why.

SOLUTION

(30 min.) Payback and NPV methods, no income taxes.

1a. Payback measures the time it will take to recoup, in the form of expected future cash
flows, the net initial investment in a project. Payback emphasizes the early recovery of cash as a
key aspect of project ranking. Some managers argue that this emphasis on early recovery of cash
is appropriate if there is a high level of uncertainty about future cash flows. Projects with shorter
paybacks give the organization more flexibility because funds for other projects become
available sooner.

Strengths
 Easy to understand

21-18
 One way to capture uncertainty about expected cash flows in later years of a project
(although sensitivity analysis is a more systematic way)

Weaknesses
 Fails to incorporate the time value of money, unless discounted payback is used
 Does not consider a project’s cash flows after the payback period

1b.

Project A

Outflow, $3,000,000
Inflow, $1,000,000 (Year 1) + $1,000,000 (Year 2) + $1,000,000 (Year 3) + $1,000,000 (Year 4)

Payback = 3 years

Project B

Outflow, $1,500,000
Inflow, $400,000 (Year 1) + $900,000 (Year 2) + $800,000 (Year 3)

($1,500 , 000−$400 ,000−$ 900 ,000 )


Payback = 2 years + $ 800 ,000 = 2.25 years

Project C

Outflow, $4,000,000
Inflow, $2,000,000 (Year 1) + $2,000,000 (Year 2) + $200,000 (Year 3) + $100,000 (Year 4)
Payback = 2 years

Payback Period
1. Project C 2 years
2. Project B 2.25 years
3. Project A 3 years

If payback period is the deciding factor, Andrews will choose only Project C (payback period = 2
years; investment = $4,000,000). Assuming that each of the projects is an all-or-nothing
investment, Andrews will have $1,000,000 left over in the capital budget, not enough to make
the $1,500,000 investment in Project B, the next best option.

2. Solution Exhibit 21-27 shows the following ranking:

NPV
1. Project B $ 207,800
2. Project A $ 169,000
3. Project C $(311,500)

21-19
3. Using NPV rankings, Projects B and A, which require a total investment of $3,000,000 +
$1,500,000 = $4,500,000, which is less than the $5,000,000 capital budget, should be funded.
This does not match the rankings based on payback period because Projects B and A have
substantial cash flows after the payback period, cash flows that the payback period ignores.

Nonfinancial qualitative factors should also be considered. For example, are there
differential worker safety issues across the projects? Are there differences in the extent of
learning that can benefit other projects? Are there differences in the customer relationships
established with different projects that can benefit Andrews Construction in future projects?

21-20
SOLUTION EXHIBIT 21-27

Present
Value Sketch of Relevant Cash Flows
Discount
Total Present Factors at
Value 10% 0 1 2 3 4
PROJECT A
Net initial invest. $(3,000,000)
1.000 $(3,000,000)

Annual cash inflow 909,000 0.909 $1,000,000


826,000 0.826 $1,000,000
751,000 0.751 $1,000,000
683,000 0.683 $1,000,000
Net present value $ 169,000

PROJECT B
Net initial invest. $(1,500,000)
1.000 $(1,500,000)

Annual cash inflow 363,600 0.909 $ 400,000


743,400 0.826 $ 900,000
600,800 0.751 $ 800,000
Net present value $ 207,800

PROJECT C
Net initial invest. $(4,000,000)
1.000 $(4,000,000)

Annual cash inflow 1,818,000


0.909 $2,000,000
1,652,000
0.826 $2,000,000
150,200 0.751 $ 200,000
68,300 0.683 $ 100,000
Net present value $ (311,500)

21-21
21-28 DCF, accrual accounting rate of return, working capital, evaluation of performance,
no income taxes. Laverty Clinic plans to purchase a new centrifuge machine for its New York
facility. The machine costs $94,000 and is expected to have a useful life of 6 years, with a
terminal disposal value of $9,000. Savings in cash operating costs are expected to be $24,900 per
year. However, additional working capital is needed to keep the machine running efficiently. The
working capital must continually be replaced, so an investment of $4,000 needs to be maintained
at all times, but this investment is fully recoverable (will be “cashed in”) at the end of the useful
life. Laverty Clinic’s required rate of return is 12%. Ignore income taxes in your analysis.
Assume all cash flows occur at year-end except for initial investment amounts. Laverty Clinic
uses straight-line depreciation for its machines.

Required:
1. Calculate net present value.
2. Calculate internal rate of return.
3. Calculate accrual accounting rate of return based on net initial investment.
4. Calculate accrual accounting rate of return based on average investment.
5. You have the authority to make the purchase decision. Why might you be reluctant to base
your decision on the DCF methods?

SOLUTION

(25–30 min.) DCF, accrual accounting rate of return, working capital, evaluation of
performance, no income taxes.

1. Present value of annuity of savings in cash operating costs


($24,900 per year for 6 years at 12%): $24,900  4.111 $102,364
Present value of $9,000 terminal disposal price of machine at
end of year 6: $9,000  0.507 4,563
Present value of $4,000 recovery of working capital at
end of year 6: $4,000  0.507 2,028
Gross present value 108,955
Deduct net initial investment:
Centrifuge machine, initial investment $94,000
Additional working capital investment 4,000 98,000
Net present value $ 10,955

2. The sequence of cash flows from the project is:

For a $98,000 initial outflow, the project generates $24,900 in cash flows at the end of each
of years one through five and $37,900 (= $24,900 + $9,000 + $4,000) at the end of year 6.

Using either a calculator or Excel, the internal rate of return for this stream of cash flows is
found to be 15.66%.

3. Accrual accounting rate of return based on net initial investment:


Net initial investment = $94,000 + $4,000
= $98,000

21-22
Annual depreciation
($94,000 – $9,000) ÷ 6 years = $14,166.67

$24,900  $14,166.67
Accrual accounting rate of return = $98,000 = 10.95%.

4. Accrual accounting rate of return based on average investment:

Net terminal cash flow = $9,000 terminal disposal price


+ $4,000 working capital recovery
= $13,000

Average investment = ($98,000 + $13,000) / 2


= $55,500

$24,900  $14,166.67
Accrual accounting rate of return = $55,500 = 19.34%.

5. If your decision is based on the DCF model, the purchase would be made because the net
present value is positive, and the 15.66% internal rate of return exceeds the 12% required rate of
return. However, you may believe that your performance may actually be measured using
accrual accounting. This approach would show a 10.95% return on the initial investment, which
is below the required rate. Your reluctance to make a “buy” decision would be quite natural
unless you are assured of reasonable consistency between the decision model and the
performance evaluation method.

21-29 New equipment purchase, income taxes. Ella’s Bakery plans to purchase a new oven
for its store. The oven has an estimated useful life of 4 years. The estimated pretax cash flows for
the oven are as shown in the table that follows, with no anticipated change in working capital.
Ella’s Bakery has a 14% after-tax required rate of return and a 35% income tax rate. Assume
depreciation is calculated on a straight-line basis for tax purposes using the initial investment in
the oven and its estimated terminal disposal value. Assume all cash flows occur at year-end
except for initial investment amounts.

21-23
Required:
1. Calculate (a) net present value, (b) payback period, and (c) internal rate of return.
2. Calculate accrual accounting rate of return based on net initial investment.

SOLUTION

(40 min.) New equipment purchase, income taxes.

1. The after-tax cash inflow per year is $29,600 ($21,600 + $8,000), as shown below:

Annual cash flow from operations $77,000


Deduct income tax payments (0.35 × $36,000) 26,950
Annual after-tax cash flow from operations $50,050

Annual depreciation on machine


[($186,000 – $6,000) ÷ 4] $45,000
Income tax cash savings from annual depreciation deductions
(0.35 × $45,000) $15,750

a. Solution Exhibit 21-29 shows the NPV computation. NPV = $9,228


b. Payback = $186,000 ÷ ($50,050 + $15,750) = 2.83 years
c. For a $186,000 initial outflow, the project generates $65,800 ($50,050 + $15,750) in after-tax
cash flows at the end of each of years one through four and an additional $6,000 at the end of
year 4.

Using either a calculator or Excel, the internal rate of return for this stream of cash flows
is found to be 16.38%.

2. Accrual accounting rate of return based on net initial investment:


Net initial investment = $186,000

Annual after-tax operating income = $65,800 - $45,000 depreciation


= $20,800

$20,800
Accrual accounting rate of return = $186,000 = 11.18%.

21-24
SOLUTION EXHIBIT 21-29
Present
Value
Total Discount
Present Factor
Value at 12% Sketch of Relevant After-Tax Cash Flows
0 1 2 3 4
1a. Initial machine
investment $(186,000) 1.000 $(186,000)
1b. Initial working
capital investment 0 1.000 $0
2a. Annual after-tax
cash flow from
operations (excl. depr.)
Year 1 43,894 0.877 $50,050
Year 2 38,488 0.769 $50,050
Year 3 33,784 0.675 $50,050
Year 4 29,630 0.592 $50,050
2b. Income tax
cash savings
from annual
depreciation
deductions
Year 1 13,813 0.877 $15,750
Year 2 12,112 0.769 $15,750
Year 3 10,631 0.675 $15,750
Year 4 9,324 0.592 $15,750
3. After-tax
cash flow from:
a. Terminal
disposal of
machine 3,552 0.592 $6,000
b. Recovery of
working capital 0 0.592 $0
Net present
value if new
machine is
purchased $ 9,228

21-30 New equipment purchase, income taxes. Walker Inc. is considering the purchase of
new equipment that will automate production and thus reduce labor costs. Walker made the
following estimates related to the new machinery:

Cost of the equipment $120,000


Reduced annual labor costs $40,000

21-25
Estimated life of equipment 5 years
Terminal disposal value $0
After-tax cost of capital 8%
Tax rate 25%

Assume depreciation is calculated on a straight-line basis for tax purposes. Assume all cash
flows occur at year-end except for initial investment amounts.

Required:
1. Calculate (a) net present value, (b) payback period, (c) discounted payback period, and (d)
internal rate of return.
2. Compare and contrast the capital budgeting methods in requirement 1.

SOLUTION

(40 min.) New equipment purchase, income taxes.

1. The after-tax cash inflow per year is $36,000 ($30,000 + $6,000), as shown below:

Annual cash flow from operations $40,000


Deduct income tax payments (0.25  $40,000) 10,000
Annual after-tax cash flow from operations $30,00

Annual depreciation on equipment ($120,000  5 years) $24,000


Income tax cash savings from annual depreciation deductions
(0.25  $24,000) $ 6,000

a. Solution Exhibit 21-30 shows the NPV computation. NPV= $23,748

An alternative approach:
Present value of 5-year annuity of $36,000 at 8%
$36,000  3.993 $143,748
Present value of cash outlay, $120,000  1.000 (120,000)
Net present value* $ 23,748

b. Payback = $120,000 ÷ $36,000


= 3.33 years

21-26
c. Discounted Payback Period

Period Cash Savings Disc Factor Discounted Cumulative Unrecovered


(8%) Cash Savings Disc Cash Investment
Savings
0 ($120,000)
1 $36,000 0.926 $33,336 $ 33,336 ($ 86,664)
2 $36,000 0.857 $30,852 $ 64,188 ($ 55,812)
3 $36,000 0.794 $28,584 $ 92,772 ($ 27,228)
4 $36,000 0.735 $26,460 $119,232 ($ 778)
5 $36,000 0.681 $24,516 $143,748

$778/$24,516 = .03
Discounted Payback Period = 4.03 years

d. For a $120,000 initial outflow, the project generates $36,000 in after-tax cash flows at the
end of each of years one through five.

Using either a calculator or Excel, the internal rate of return for this stream of cash flows is
found to be 15.24%.

2. Both the net present value and internal rate of return methods use the discounted cash
flow approach in which all expected future cash inflows and outflows of a project are measured
as if they occurred at a single point in time. The net present value approach computes the surplus
generated by the project in today’s dollars while the internal rate of return attempts to measure its
effective return on investment earned by the project.

The payback method, by contrast, considers nominal cash flows (without discounting)
and measures the time at which the project’s expected future cash inflows recoup the net initial
investment in a project. The payback method thus ignores the profitability of the project’s entire
stream of future cash flows. The discounted payback method shares this last defect, but looks at
the time taken to recoup the initial investment based on the discounted present value of cash
inflows. The two payback methods are becoming increasingly important in the global economy.
When the local environment in an international location is unstable and therefore highly risky for
a potential investment, a company would likely pay close attention to the payback period for
making its investment decision. In general, the more unstable the environment, the shorter the
payback period desired.

21-27
SOLUTION EXHIBIT 21-30
Present Value
Total Discount
Present Factors
Value At 8% Sketch of Relevant After-Tax Cash Flows
0 1 2 3 4 5
1a. Initial $(120,000)
equipment
investment 1.000 $(120,000)
1b. Initial working $0
capital investment 0 1.000
2a. Annual after-
tax cash flow from
operations (excl. depr.)
Year 1 27,780 0.926 $30,000
Year 2 25,710 0.857 $30,000
Year 3 23,820 0.794 $30,000
Year 4 22,050 0.735 $30,000
Year 5 20,430 0.681 $30,000
2b Income tax cash
savings from annual
depreciation
deductions
Year 1 5,556 0.926 $6,000
Year 2 5,142 0.857 $6,000
Year 3 4,764 0.794 $6,000
Year 4 4,410 0.735 $6,000
Year 5 4,086 0.681 $6,000
3. After-tax cash
flow from:
a. Terminal disposal 0.681
of
equipment 0 $0
b. Recovery of 0
working capital 0.681 $0
Net present value if
new equipment is
purchased $23,748

21-28
21-31 Project choice, taxes. Klein Dermatology is contemplating purchasing new laser
therapy equipment. This new equipment would cost $300,000 to purchase and $20,000 for
installation. Klein estimates that this new equipment would yield incremental margins of
$98,000 annually due to new client services but would require incremental cash maintenance
costs of $10,000 annually. Klein expects the life of this equipment to be 5 years and estimates a
terminal disposal value of $20,000.
Klein has a 25% income tax rate and depreciates assets on a straight-line basis (to terminal
value) for tax purposes. The required rate of return on investments is 10%.

Required:
1. What is the expected increase in annual net income from investing in the improvements?
2. Calculate the accrual accounting rate of return based on average investment.
3. Is the project worth investing in from an NPV standpoint?
4. Suppose the tax authorities are willing to let Klein depreciate the project down to zero over
its useful life. If Klein plans to liquidate the project in 5 years, should it take this option?
Quantify the impact of this choice on the NPV of the project.

SOLUTION

(25 min.) Project choice, taxes.

1. The after-tax cash inflow per year is $81,000 ($66,000 + $15,000), as shown below:

Annual cash flow from operations ($98,000 – $10,000) $88,000


Deduct income tax payments (0.25  $88,000) 22,000
Annual after-tax cash flow from operations $66,000

Annual depreciation on new equipment


[($320,000 – 20,000]  5 years) $60,000
Income tax cash savings from annual depreciation deductions
(0.25  $60,000) $ 15,000

The total after-tax cash inflow is thus $66,000 + $15,000 = $81,000.

The expected increase in net annual income is $21,000, the difference between the after-
tax cash inflow of $81,000 and the annual depreciation of $60,000.

This can also be computed directly as follows:

Incremental margins $98,0


Deduct incremental cash expenses 10,00
Deduct depreciation on equipment 60,00
Pre-tax incremental income $28,000
Incremental tax (0.25  $28,000) 7,000
After-tax incremental income $ 21,000

21-29
$320,000 +$20,000
2. The average level of investment in the project is 2 = $170,000.
The after-tax incremental income from the project (from requirement 1) is $21,000.

The accrual accounting rate of return on average investment is therefore

$21,000
$170,000 = 12.35%.

3. The equipment is not worth investing in from an NPV standpoint. Its NPV is $(509),
calculated as follows:

Present value of 5-year annuity of $81,000 at 10%


$81,000  3.791 $ 307,071
Present value of $20,000 disposal value at end of 5 years
$20,000  0.621 12,420
Present value of cash outlays, $320,000  1.000 (320,000)
Net present value $ (509)

4. The first effect of the change in depreciation policy is that Klein can depreciate $64,000
of the asset each period rather than $60,000. This will increase its income tax cash
savings from annual depreciation to $16,000 (0.25 × $64,000) and increase the overall
after-tax annual cash inflow to $82,000 ($66,000 + $16,000).

The second effect is that when Klein disposes of the equipment at the end of year 5 for
$20,000, it owes tax on the gain in sale of the asset (which has now been depreciated to
$0). It will receive a net cash flow of $15,000 (0.75 × $20,000) at that time.

The project is now worth investing in from an NPV standpoint. Its NPV is $177,
calculated as follows:

Present value of 5-year annuity of $82,000 at 10%


$82,000  3.791 $ 310,862
Present value of $15,000 net disposal value at end of 5 years
$15,000  0.621 9,315
Present value of cash outlays, $100,000  1.000 (320,000)
Net present value $ 177

From Klein’s standpoint, the new depreciation policy is clearly better and leads to a
change in its decision regarding acceptance of the project. In general, for purposes of
capital budgeting, any policy that permits a firm to accelerate depreciations or write-
downs is better, even if it entails paying taxes on disposal later, because of the time value
of receiving the cash flows earlier.

21-30
21-32 Customer value. Ortel Telecom sells telecommunication products and services to a
variety of small businesses. Two of Ortel’s key clients are Square and Cloudburst, both fast-
growing technology start-ups located in New York City. Ortel has compiled information
regarding its transactions with Square and Cloudburst for 2017, as well as its expectations
regarding their interactions over the next 3 years:

Ortel’s transactions with Square and Cloudburst are in cash. Assume that they occur at year-end.
Ortel is headquartered in the Cayman Islands and pays no income taxes. The owners of Ortel
insist on a required rate of return of 12%.

Required:
1. What is the expected net cash flow from Square and Cloudburst for the next 3 years?
2. Based on the net present value from cash flows over the next 3 years, is Cloudburst or Square
a more valuable customer for Ortel?
3. Cloudburst threatens to switch to another supplier unless Ortel gives a 10% price reduction
on all sales starting in 2018. Calculate the 3-year NPV of Cloudburst after incorporating the
10% discount. Should Ortel continue to transact with Cloudburst? What other factors should
it consider before making its final decision?

SOLUTION

(40 min.) Customer value.

1.
Annual
Increase
Square s* 2017 2018 2019 2020
$567,00 $601,02 $637,08 $675,30
Cash Revenues 6% 0 0 1 6
364,80 40 422,3
Cash Expenses 5% 0 383,040 2,192 02

$202,20 $217,98 $234,88 $


Net Cash Flows 0 0 9 253,004

Cloudburst Annual 2017 2018 2019 2020

21-31
Increase
s*
$3,510,0 $3,703,0 $3,906,7 $4,121,5
Cash Revenues 5.5% 00 50 18 87
3,060,0 3,197,7 3,341,5 3,491,9
Cash Expenses 4.5% 00 00 97 68
$ $ $ $
Net Cash Flows 450,000 505,350 565,121 629,619

*
Given in the problem.

2.
    Square Cloudburst

PV Factor Net Cash Present Net Cash Present


Year for 12% Flows Value Flows Value
$ $
2 5
1 0
7 5
, ,
9 3
8 $194,65 5
2018 0.893 0 6 0 $451,278
2 5
3 6
4 5
, ,
8 1
8 2
2019 0.797 9 187,207 1 450,401
2 6
5 2
3 9
, ,
0 6
0 1 448,28
2020 0.712 4 180,139 9 9
$562,00 $1,349,96
      2 8
           

Based on NPV at 12%, Cloudburst is the more valuable customer.

3. Assuming a 10% discount on the revenues for Cloudburst calculated in requirement 1, we


have

21-32
Annual
Increase
Cloudburst s 2017 2018 2019 2020
$3,510,00 $3,516,04
Cash Revenues* 5.5% 0 $3,332,745 6 $3,709,428
3,060,00 3,341,59 3,491,96
Cash Expenses 4.5% 0 3,197,700 7 8
$ $ $
Net Cash Flows 450,000 $ 135,045 174,449 217,460
*
Cloudburst’s revenue from requirement 1 reduced by 10% each year from 2018 onwards.

Net present value if revenues are reduced by 10% each year relative to original estimates:

    Cloudburst
PV Factor Net Cash Present
Year for 12% Flows Value
$
1
3
5,
0
4 $120,59
2018 0.893 5 5
1
7
4,
4
4
2019 0.797 9 139,036
2
1
7,
4
6 154,83
2020 0.712 0 2
$414,46
      3
       

The 10% discount and reduced subsequent annual revenue reduces the NPV from
$1,349,968 to $414,463. The NPV is still positive and so Ortel should continue to sell to
Cloudburst Constructors. However, this is almost a 70% drop in NPV from Cloudburst, and it
makes Square now the more profitable customer.

21-33
Ortel should consider whether the price discount demanded by Cloudburst needs to be met
in its entirety to keep the account. The implication of meeting the full demand is that the account
is minimally profitable. A serious concern is whether Squrare will also demand comparable price
discounts if Cloudburst’s demands are met. This could result in large reductions in the NPVs of
all of Ortel’s customers.
Ortel should also consider the reliability of the growth estimates used in computing the
NPVs. Are the predicted differences in revenue growth rates based on reliable information? Many
revenue growth estimates by salespeople turn out to be overestimates or occur over a longer time
period than initially predicted.

21-33 Selling a plant, income taxes. (CMA, adapted) The Cook Company is a national
portable building manufacturer. Its Benton plant will become idle on December 31, 2017. Mary
Carter, the corporate controller, has been asked to look at three options regarding the plant:

■ Option 1: The plant, which has been fully depreciated for tax purposes, can be sold
immediately for $750,000.
■ Option 2: The plant can be leased to the Timber Corporation, one of Cook’s suppliers, for 4
years. Under the lease terms, Timber would pay Cook $175,000 rent per year (payable at
year-end) and would grant Cook a $60,000 annual discount from the normal price of lumber
purchased by Cook. (Assume that the discount is received at year-end for each of the 4
years.) Timber would bear all of the plant’s ownership costs. Cook expects to sell this plant
for $250,000 at the end of the 4-year lease.
■ Option 3: The plant could be used for 4 years to make porch swings as an accessory to be
sold with a portable building. Fixed overhead costs (a cash outflow) before any equipment
upgrades are estimated to be $22,000 annually for the 4-year period. The swings are expected
to sell for $45 each. Variable cost per unit is expected to be $22. The following production
and sales of swings are expected: 2018, 12,000 units; 2019, 18,000 units; 2020, 15,000 units;
2021, 8,000 units. In order to manufacture the swings, some of the plant equipment would
need to be upgraded at an immediate cost of $180,000. The equipment would be depreciated
using the straight-line depreciation method and zero terminal disposal value over the 4 years
it would be in use. Because of the equipment upgrades, Cook could sell the plant for
$320,000 at the end of 4 years. No change in working capital would be required.

Required:
Cook Company treats all cash flows as if they occur at the end of the year, and uses an after-tax
required rate of return of 8%. Cook is subject to a 30% tax rate on all income, including capital
gains.

1. Calculate net present value of each of the options and determine which option Cook should
select using the NPV criterion.
2. What nonfinancial factors should Cook consider before making its choice?

SOLUTION

(60 min.) Selling a plant, income taxes.

21-34
1. Option 1
Current disposal price $750,000
Deduct current book value 0
Gain on disposal 750,000
Deduct 30% tax payments 225,000
Net present value $525,000

Option 2
Cook receives three sources of cash inflows:
a. Rent. Four annual payments of $175,000. The after-tax cash inflow is:
$175,000 × (1 – 0.30) = $122,500 per year

b. Discount on material purchases, payable at year-end for each of the four years: $60,000
The after-tax cash inflow is: $60,000 × (1 – 0.30) = $42,000

c. Sale of plant at year-end 2018. The after-tax cash inflow is:


$250,000 × (1 – 0.30) = $175,000

Present Value
Total Discount
Present Factors at
Value 8% Sketch of Relevant After-Tax Cash Flows
0 1 2 3 4

1. Rent
$113,435 0.926 $122,500
104,983 0.857 $122,500
97,265 0.794 $122,500
90,038 0.735 $122,500

2. Discount on
Purchases $38,892 0.926 $42,000
35,994 0.857 $42,000
33,348 0.794 $42,000
30,870 0.735 $42,000

3. Sale of plant $128,625 0.735 $175,000

Net present value $673,450

Option 3

Contribution margin per swing:


Selling price $45.00
Variable costs 22.00
Contribution margin $23.00

2018 2019 2020 2021

21-35
Contribution margin
$23.00 × 12,000; 18,000;
15,000; 8,000 $276,000 $414,000 $345,000 $184,000
Fixed overhead (cash) costs 22,000 22,000 22,000 22,000
Annual cash flow from operations 254,000 392,000 323,000 162,000
Income tax payments (30%) 76,200 117,600 96,900 48,600
After-tax cash flow from
operations (excl. depcn.) $177,800 $274,400 $226,100 $113,400

Depreciation: $180,000 ÷ 4 = $45,000 per year


Income tax cash savings from depreciation deduction: $45,000 × 0.30 = $13,500 per year
Sale of plant at end of 2018: $320,000 × (1 – 0.30) = $224,000
Solution Exhibit 21-33 presents the NPV calculations: NPV = $692,029

SOLUTION EXHIBIT 21-33


Present Value
Total Discount
Present Factors at
Value 10% Sketch of Relevant After-Tax Cash Flows
2017 2018 2019 2020 2021
1a. Initial plant equipment $(
upgrade investment 18
0,0
00
) 1.000 $(180,000)
1b. Initial working capital
investment 0 1.000 $0
2a. Annual after-tax cash
flow from operations
(excluding depreciation
effects)
Year 1 1
6
4,
6
4
3 0.926 $177,800
Year 2 2
3
5,
1
6
1 0.857 $274,400
Year 3 1
7
9,
5
2
3 0.794 $226,100

21-36
Year 4 8
3,
3
4
9 0.735 $113,400
2b. Income tax cash savings
from annual depreciation
deductions
Year 1 1
2,
5
0
1 0.926 $13,500
Year 2 1
1,
5
7
0 0.857 $13,500
Year 3 1
0,
7
1
9 0.794 $13,500
Year 4 9,
9
2
3 0.735 $13,500
3. After-tax cash flow
From
a. Terminal disposal 1
of plant 6
4,
6
4
0 0.735 $224,000
b. Recovery of working
capital 0 0.735 $0
$
6
9
2,
0
2
Net present value 9

Option 3 has the highest NPV:


NPV
Option 1 $525,000
Option 2 $673,450
Option 3 $692,029

2. Nonfinancial factors that Cook should consider include the following:


 Option 1 gives Cook immediate liquidity that it can use for other projects.

21-37
 Option 2 has the advantage of providing Cook a closer relationship with one of its
suppliers. However, it limits Cook’s flexibility if Timber Corporation’s quality is not
comparable to that of competitors.
 Option 3 has Cook entering a new line of business. If this line of business is
successful, it could be expanded to cover other accessories that could be sold along
with the portable buildings. The risks of selling the predicted number of porch swings
should also be considered.

21-34 Equipment replacement, no income taxes. Dublin Chips is a manufacturer of prototype


chips based in Dublin, Ireland. Next year, in 2018, Dublin Chips expects to deliver 615 prototype
chips at an average price of $95,000. Dublin Chips’ marketing vice president forecasts growth of
65 prototype chips per year through 2024. That is, demand will be 615 in 2018, 680 in 2019, 745
in 2020, and so on.
The plant cannot produce more than 585 prototype chips annually. To meet future demand,
Dublin Chips must either modernize the plant or replace it. The old equipment is fully
depreciated and can be sold for $4,200,000 if the plant is replaced. If the plant is modernized, the
costs to modernize it are to be capitalized and depreciated over the useful life of the updated
plant. The old equipment is retained as part of the modernize alternative. The following data on
the two options are available:

Modernize Repl
ace
Initial investment in 2018 $35,300,000 $66,300,000
Terminal disposal value in 2024 $ 7,500,000 $16,000,000
Useful life 7 years 7 years
Total annual cash operating costs per prototype chip $78,500 $66,000

Dublin Chips uses straight-line depreciation, assuming zero terminal disposal value. For
simplicity, we assume no change in prices or costs in future years. The investment will be made
at the beginning of 2018, and all transactions thereafter occur on the last day of the year. Dublin
Chips’ required rate of return is 14%.
There is no difference between the modernize and replace alternatives in terms of required
working capital. Dublin Chips has a special waiver on income taxes until 2024.

Required:
1. Sketch the cash inflows and outflows of the modernize and replace alternatives over the
2018–2024 period.
2. Calculate the payback period for the modernize and replace alternatives.
3. Calculate net present value of the modernize and replace alternatives.
4. What factors should Dublin Chips consider in choosing between the alternatives?

SOLUTION

(60 min.) Equipment replacement, no income taxes.

21-38
1. Cash flows for modernizing alternative:

Net Cash Initial Sale of Equip.


Year Units Sold Contributions Investments at Termination
(1) (2) (3) = (2) × $16,500a (4) (5)

Jan. 1, 2018 –– –– $(35,300,000) ––


Dec. 31, 2018 615 $10,147,500
Dec. 31, 2019 680 11,220,000
Dec. 31, 2020 745 12,292,500
Dec. 31, 2021 810 13,365000
Dec. 31, 2022 875 14,437,500
Dec. 31, 2023 940 15,510000
Dec. 31, 2024 1,005 16,582,500 $7500000
a
$95000 – $78,500 = $16,500 cash contribution per prototype.

Cash flows for replacement alternative:


Net Cash Initial Sale of Equip.
Year Units Sold Contributions Investments
(1) (2) (3) = (2) × $29,000b (4) (5)

Jan. 1, 2018 –– –– $(66,300,000) $ 4,200,000

Dec. 31, 2018 615 $17,835,000


Dec. 31, 2019 680 19,720,000
Dec. 31, 2020 745 21,605,000
Dec. 31, 2021 810 23,490000
Dec. 31, 2022 875 25,375000
Dec. 31, 2023 940 27,260000
Dec. 31, 2024 1,005 29,145,000 $16000000
b
$95000 – $66000 = $29000 cash contribution per prototype.

2. Payback period calculations for modernizing alternative:

Cumulative Net Initial Investment


Year Cash Inflow Cash Inflow Unrecovered at End of Year
(1) (2) (3) (4)

Jan. 1, 2018 –– –– $35,300,000


Dec. 31, 2018 $10,147,500 $10,147,500 25,152500
Dec. 31, 2019 11,220000 21,367500 13,932500
Dec. 31, 2020 12,292,500 33,660000 1,640000
Dec. 31, 2021 13,365,000 47,025,000 0

21-39
Payback
= 3 + ($1,640,000 ÷ $13,365,000)
= 3.12 years

Payback period calculations for replace alternative:

Cumulative Net Initial Investment


Year Cash Inflow Cash Inflow Unrecovered at End of Year
(1) (2) (3) (4)

Jan. 1, 2018 –– –– $62,100,000


Dec. 31, 2018 $17,835000 $17,835000 44,265000
Dec. 31, 2019 19,720000 37,555000 24,545000
Dec. 31, 2020 21,605000 59,160000 2,940000
Dec. 31, 2021 23,490000 82,650,000 0

Payback = 3 + ($2,940,000 ÷ $23,490,000)


= 3.13 years

3. Modernizing alternative:

Present Value
Discount Factors Net Cash Present
Year At 14% Flow Value
Jan. 1, 2018 1.000 $(35,300000) $(35,300,000)
Dec. 31, 2018 0.877 10,147,500 8,899,358
Dec. 31, 2019 0.769 11,220000 8,628,180
Dec. 31, 2020 0.675 12,292,500 8,297,438
Dec. 31, 2021 0.592 13,365,000 7,912,080
Dec. 31, 2022 0.519 14,437,500 7,493,063
Dec. 31, 2023 0.456 15,510000 7,072,560
Dec. 31, 2024 0.400 24,082,500 9,633,000
Total $22,635,679

Replace Alternative:

Present Value
Discount Factors Net Cash Present
Year At 14% Flow Value
Jan. 1, 2018 1.000 $(62,100000) $(62,100,000)
Dec. 31, 2018 0.877 17,835,000 15,641,295
Dec. 31, 2019 0.769 19,720000 15,164,680
Dec. 31, 2020 0.675 21,605,000 14,583,375
Dec. 31, 2021 0.592 23,490,000 13,906,080
Dec. 31, 2022 0.519 25,375,000 13,169,625
Dec. 31, 2023 0.456 27,260000 12,430,560

21-40
Dec. 31, 2024 0.400 45,145,000 18,058,000
Total $40,853,615

4. Using the payback period, the modernize alternative is preferred, albeit by a very slight
margin, to the replace alternative. On the other hand, the replace alternative has a significantly
higher NPV than the modernize alternative and so should be preferred. Of course, the NPV
amounts are based on best estimates of cash flows going out into the future. Dublin Chips should
examine the sensitivity of the NPV amounts to variations in the estimates.

Nonfinancial qualitative factors should also be considered. These could include the
quality of the prototypes produced by the modernize and replace alternatives. These alternatives
may differ in capacity and their ability to meet surges in demand beyond the estimated amounts.
The alternatives may also differ in how workers increase their shop floor-capabilities. Such
differences could provide labor force externalities that can be the source of future benefits to
Dublin Chips.

21-35 Equipment replacement, income taxes (continuation of 21-34). Assume the same facts
as in Problem 21-34, except that the plant is located in Buffalo, New York. Dublin Chips has no
special waiver on income taxes. It pays a 35% tax rate on all income. Proceeds from sales of
equipment above book value are taxed at the same 35% rate.

Required:
1. Sketch the after-tax cash inflows and outflows of the modernize and replace alternatives over
the 2018–2024 period.
2. Calculate the net present value of the modernize and replace alternatives.
3. Suppose Dublin Chips is planning to build several more plants. It wants to have the most
advantageous tax position possible. Dublin Chips has been approached by Spain, Malaysia,
and Australia to construct plants in their countries. Use the data in Problem 21-34 and this
problem to briefly describe in qualitative terms the income tax features that would be
advantageous to Dublin Chips.

SOLUTION

(40 min.) Equipment replacement, income taxes (continuation of 21-34).

1. & 2. Income tax rate = 35%

Modernize Alternative

Annual depreciation:
$35,300000  7 years = $5042857.14 a year.

Income tax cash savings from annual depreciation deductions:


$5042,857.14  0.35 = $1765,000 a year.

21-41
Terminal disposal of equipment = $7500000.

After-tax cash flow from terminal disposal of equipment:


$7500000  0.65 = $4,875000.

The NPV components are:

a. Initial investment: NPV


Jan. 1, 2018 $(35,300000)  1.000 $(35,300000)

b. Annual after-tax cash flow from operations


(excluding depreciation):
Dec. 31, 2018 10,147500  0.65  0.877 $5,784,582
2019 11,220000  0.65  0.769 5,608,317
2020 12,292,500  0.65  0.675 5,393,334
2021 13,365000  0.65  0.592 5,142,852
2022 14,437,500  0.65  0.519 4,870,491
2023 15,510000  0.65  0.456 4,597,164
2024 16,582,500  0.65  0.400 4,311,450

c. Income tax cash savings from annual depreciation


deductions ($1765,000 each year for 7 years):
$1765,000  4.288 7,568,320

d. After-tax cash flow from terminal sale of equipment:


$4,875,000  0.400 1,950,000

Net present value of modernize alternative $ 9,926,510

Replace alternative

Initial machine replacement = $66,300,000

Sale on Jan. 1, 2018, of equipment = $4,200,000

After-tax cash flow from sale of old equipment: $4,200,000  0.65 = $2,730,000

Net initial investment: $66,300,000  $2,730,000 = $63,570,000

Annual depreciation: $66,300,000  7 years = $9,471,428.57 a year

Income-tax cash savings from annual depreciation deductions: $9,471,428.57  0.35 =


$3,315,000

After-tax cash flow from terminal disposal of equipment: $16,000,000  0.65 = $10,400,000

21-42
The NPV components of the replace alternative are:

a. Net initial investment


Jan. 1, 2018 $(63,570,000)  1.000 $(63,570

b. Annual after-tax cash flow from operations (excluding depreciation)


Dec. 31, 2018 $17,835,000  0.65  0.877 $10,166,842
2019 19,720,000  0.65  0.769 9,857,042
2020 21,605,000  0.65  0.675 9,479,194
2021 23,490,000  0.65  0.592 9,038,952
2022 25,375,000  0.65  0.519 8,560,256
2023 27,260,000  0.65  0.456 8,079,864
2024 29,145,000  0.65  0.400 7,577,700

c. Income tax cash savings from annual depreciation deductions


($3,315,000 each year for 7 years) $3,315,000  4.288 14,214,720

d. After-tax cash flow from terminal sale of equipment, $10,400,000 4,160,000


 0.400

Net present value of replace alternative $17,564,570

On the basis of NPV, Dublin Chips should replace rather than modernize the equipment.

3. Dublin Chips would prefer to:


a. have lower tax rates,
b. have revenue exempt from taxation,
c. recognize taxable revenues in later years rather than earlier years,
d. recognize taxable cost deductions greater than actual outlay costs, and
e. recognize cost deductions in earlier years rather than later years (including
accelerated amounts in earlier years).

21-36 DCF, sensitivity analysis, no income taxes. (CMA, adapted) Sentax Corporation is an
international manufacturer of fragrances for women. Management at Sentax is considering
expanding the product line to men’s fragrances. From the best estimates of the marketing and
production managers, annual sales (all for cash) for this new line are 2,000,000 units at $100 per
unit; cash variable cost is $50 per unit; and cash fixed costs are $18,000,000 per year. The
investment project requires $100,000,000 of cash outflow and has a project life of 4 years.
At the end of the 4-year useful life, there will be no terminal disposal value. Assume all cash
flows occur at year-end except for initial investment amounts.
Men’s fragrance is a new market for Sentax, and management is concerned about the
reliability of the estimates. The controller has proposed applying sensitivity analysis to selected
factors. Ignore income taxes in your computations. Sentax’s required rate of return on this
project is 16%.

Required:

21-43
1. Calculate the net present value of this investment proposal.
2. Calculate the effect on the net present value of the following two changes in assumptions.
(Treat each item independently of the other.)
a. 20% reduction in the selling price
b. 20% increase in the variable cost per unit
3. Discuss how management would use the data developed in requirements 1 and 2 in its
consideration of the proposed capital investment.

SOLUTION

(20 min.) DCF, sensitivity analysis, no income taxes.

1. Revenues, $100 × 2,000,000 $200,000,000


Variable cash costs, $50 × 2,000,000 100,000,000
Cash contribution margin 100,000,000
Fixed cash costs 18,000,000
Cash inflow from operations $ 82,000,000

Net present value:


Cash inflow from operations: $82,000,000 × 2.798 $229,436,000
Cash outflow for initial investment (100,000,000)
Net present value $129,436,000

2a. 20% reduction in selling prices:


Revenues, $80 × 2,000,000 $160,000,000
Variable cash costs, $50 × 2,000,000 100,000,000
Cash contribution margin 60,000,000
Fixed cash costs 18,000,000
Cash inflow from operation $ 42,000,000

Net present value:


Cash inflow from operations: $42,000,000 × 2.798 $117,516,000
Cash outflow for initial investment (100,000,000)
Net present value $ 17,516,000

b. 20% increase in the variable cost per unit:


Revenues, $100 × 2,000,000 $200,000,000
Variable cash costs, $60 × 2,000,000 120,000,000
Cash contribution margin 80,000,000
Fixed cash costs 18,000,000
Cash inflow from operations $ 62,000,000

Net present value:


Cash inflow from operations: $62,000,000 × 2.798 $173,476,000
Cash outflow for initial investment (100,000,000)
Net present value $ 73,476,000

21-44
3. Sensitivity analysis enables management to see those assumptions for which input
variations have sizable impact on NPV. Extra resources could be devoted to getting more
informed estimates of those inputs with the greatest impact on NPV, in this case the potential
reduction in selling prices.

Sensitivity analysis also enables management to have contingency plans in place if the
assumptions underlying the analysis are not met.

21-37 NPV and AARR, goal-congruence issues. Liam Mitchell, a manager of the Plate
Division for the Harvest Manufacturing company, has the opportunity to expand the division by
investing in additional machinery costing $495,000. He would depreciate the equipment using the
straight-line method and expects it to have no residual value. It has a useful life of 9 years. The
firm mandates a required after-tax rate of return of 14% on investments. Liam estimates annual
net cash inflows for this investment of $130,000 before taxes and an investment in working
capital of $5,000 that will be returned at the project’s end. Harvest’s tax rate is 30%.

Required:
1. Calculate the net present value of this investment.
2. Calculate the accrual accounting rate of return based on net initial investment for this project.
3. Should Liam accept the project? Will Liam accept the project if his bonus depends on
achieving an accrual accounting rate of return of 14%? How can this conflict be resolved?

SOLUTION

(30–35 min.) NPV and AARR, goal-congruence issues.

1.
Annual cash flow from operations $130,000
Income tax payments (30%) 39,000
Annual after-tax cash flow from operations (excl. deprn.) $ 91,000

Depreciation: $495,000 ÷ 9 = $55,000 per year


Income-tax cash savings from depreciation deduction: $55,000 × 0.30 = $16,500 per year

The present value of an annuity of $1 per year for 9 years discounted at 14% = 4.946.
So, present value of annual cash flows = ($91,000 + $16,500) × 4.946 = $531,695

Net initial investment = $(495,000) + $(5,000) = $(500,000)

Present value of working capital recovery = $5,000 × 0.308 = $1,540

Net present value of project = $(500,000) + $531,695 + $1,540 = $33,235

21-45
2. Accrual accounting rate of return (AARR): The accrual accounting rate of return takes the
annual accrual net income after tax and divides by the initial investment to get a return.

Incremental net operating income excluding depreciation $130,000


Less: Depreciation expense ($495,000 ÷ 9) 55,000
Income before tax 75,000
Income tax expense (at 30%) 22,500
Net income per period $ 52,500

AARR = $52,500 ÷ $500,000 = 10.50%.

3. Liam will not accept the project if he is being evaluated on the basis of accrual accounting
rate of return, because the project does not meet the 14% threshold above which Liam earns a
bonus. Liam should accept the project if he wants to act in the firm’s best interest because the
NPV is positive, implying that, based on the cash flows generated, the project exceeds the firm’s
required 14% rate of return. Thus, Liam will turn down an acceptable long-run project to avoid a
poor evaluation based on the measure used to evaluate his performance. To remedy this, the firm
could evaluate Liam instead on a project-by-project basis, by looking at how well he achieves the
cash flows forecasted when he chose to accept the project.

21-38 Payback methods, even and uneven cash flows. Sage Laundromat is trying to enhance
the services it provides to customers, mostly college students. It is looking into the purchase of
new high-efficiency washing machines that will allow for the laundry’s status to be checked via
smartphone.
Sage estimates the cost of the new equipment at $159,000. The equipment has a useful life of
9 years. Sage expects cash fixed costs of $80,000 per year to operate the new machines, as well
as cash variable costs in the amount of 5% of revenues. Sage evaluates investments using a cost
of capital of 10%.

Required:
1. Calculate the payback period and the discounted payback period for this investment,
assuming Sage expects to generate $140,000 in incremental revenues every year from the
new machines.
2. Assume instead that Sage expects the following uneven stream of incremental cash revenues
from installing the new washing machines:

Based on this estimated revenue stream, what are the payback and discounted payback
periods for the investment?

SOLUTION

21-46
(30 min.) Payback methods, even and uneven cash flows.

Payback problem:

1. Annual revenue $140,000


Annual costs
Fixed $80,000
Variable 7,000 87,000
Net annual cash inflow $ 53,000

Payback period = Investment / Net cash inflows = $159,000 / $53,000 = 3 years

Discounted Payback Period with even cash flows:

Peri Year Cash Fixed Variabl Net Disc Discounted Cumulative Unrecovered
Revenues Costs e Costs Cash Facto Cash Disc. Cash Investment
Inflows r Savings Savings
(10%)
0 $159,000
1 $140,000 $80,000 $7,000 $53,000 .909 $48,177 $ 48,177 $110,823
2 $140,000 $80,000 $7,000 $53,000 .826 $43,778 $ 91,955 $ 67,045
3 $140,000 $80,000 $7,000 $53,000 .751 $39,803 $131,758 $ 27,242
4 $140,000 $80,000 $7,000 $53,000 .683 $36,199 $167,957 $ 0
5 $140,000 $80,000 $7,000 $53,000 .621 $32,913
6 $140,000 $80,000 $7,000 $53,000 .564 $29,892
7 $140,000 $80,000 $7,000 $53,000 .513 $27,189
8 $140,000 $80,000 $7,000 $53,000 .467 $24,751
9 $140,000 $80,000 $7,000 $53,000 .424 $22,472

In year 4, $27,242/$36,199 = .75


Discounted Payback Period = 3.75 years

2.
Cash Fixed Cash
Revenue Costs Variable Costs Net Cash Inflows Cumulative
Year (1) (2) (3) (4) = (1) − (2) − (3) Amounts
1 $ 90,000 $ 80,000 $ 4,500 $ $
5,500 5,
50

21-47
0
39
34,0 ,5
2 120,000 80,000 6,000 00 00
78
38,75 ,2
3 125,000 80,000 6,250 0 50
79
,0
4 85,000 80,000 4,250 750 00
14
1,
62,50 50
5 150,000 80,000 7,500 0 0
26
1,
119,5 00
6 210,000 80,000 10,500 00 0
30
4,
43,50 50
7 130,000 80,000 6,500 0 0
35
7,
53,00 50
8 140,000 80,000 7,000 0 0
45
8,
100,50 00
9 190,000 80,000 9,500 0 0

The cumulative amount exceeds the initial $159,000 investment for the first time at the end of
year 6. So, payback happens in year 6.

Using linear interpolation, a more precise measure is that payback happens at:

5 years + ($159,000 - $141,500)/$119,500=5.15 years

Discounted Payback Period with uneven cash flows:

Year Cash Fixed Variable Net Cash Disc Discounted Cumulative Unrecovered
Revenues Costs Costs Inflows Factor Cash Disc. Cash Investment
(10%) Savings Savings
0 $159,000
1 $ $80,000 $ 4,500 $ 5,500 .909 $ 5,000 $ 5,000 154,000

21-48
9
0,
0
0
0
2 120,000 80,000 6,000 34,000 .826 28,084 33,084 125,916
3 125,000 80,000 6,250 38,750 .751 29,101 62,185 96,815
4 85,000 80,000 4,250 750 .683 512 62,697 96,303
5 150,000 80,000 7,500 62,500 .621 38,813 101,510 57,490
6 210,000 80,000 10,500 119,500 .564 67,398 168,908 0
7 130,000 80,000 6,500 43,500 .513 22,316 191,224
8 140,000 80,000 7,000 53,000 .467 24,751 215,975
9 190,000 80,000 9,500 100,500 .424 42,612 258,587

Discounted payback period = 5 years + $57,490/$67,398 = 5.85 years

21-39 Replacement of a machine, income taxes, sensitivity. (CMA, adapted) The Kuhl
Brothers own a frozen custard ice cream shop. The brothers currently are using a machine that
has been in use for the last 4 years. On January 1, 2017, the Kuhl Brothers are considering
buying a new machine to make their frozen custard. The Kuhl Brothers have two options: (1)
continue using the old freezing machine or (2) sell the old machine and purchase a new freezing
machine. The seller of the new machine is not interested in a trade-in of Kuhl’s old machine. The
following information has been obtained:

Required:
The Kuhl Brothers are subject to a 25% income tax rate. Any gain or loss on the sale of machines
is treated as an ordinary tax item and will affect the taxes paid by the Kuhl Brothers in the year in
which it occurs. The Kuhl Brothers have an after-tax required rate of return of 8%. Assume all
cash flows occur at year-end except for initial investment amounts.

21-49
1. The Kuhl Brothers ask you whether they should buy the new machine. To help in your
analysis, calculate the following:
a. One-time after-tax cash effect of disposing of the old machine on January 1, 2017
b. Annual recurring after-tax cash operating savings from using the new machine (variable
and fixed)
c. Cash tax savings due to differences in annual depreciation of the old machine and the
new machine
d. Difference in after-tax cash flow from terminal disposal of new machine and old machine
2. Use your calculations in requirement 1 and the net present value method to determine
whether the Kuhl Brothers should continue to use the old machine or acquire the new
machine.
3. How much more or less would the recurring after-tax cash operating savings of the new
machine need to be for the Kuhl Brothers to earn exactly the 8% after-tax required rate of
return? Assume that all other data about the investment do not change.

SOLUTION

(40 min.) Replacement of a machine, income taxes, sensitivity.

1a. Original cost of old machine: $180,000


Depreciation taken during the first 3 years
{[($180,000 – $13,500) ÷ 9]  4} 74,000
Book value 106,000
Current disposal price: 75,000
Loss on disposal $ 31,000
Tax rate × 0.25
Tax savings from loss on current disposal of old machine $ 7,750

Total after-tax cash effect of disposal = $75,000 + $7,750 = $82,750

1b. Difference in recurring after-tax variable cash-operating savings, with 25% tax rate:
($0.50 – $0.40)  (240,000)  (1– 0.25) = $18,000 (in favor of new machine)

Difference in after-tax fixed cost savings, with 25% tax rate:


($12,000 – $8,000)  (1 – 0.25) = $3,000 (in favor of new machine)

1c.
Old New
Mach Mach
ine ine
Initial machine investment $180,000 $225,000
Terminal disposal price at end of useful life 13,500 20,000
Depreciable base $166,500 $205,000
Annual depreciation using

21-50
straight-line (9-year life) $ 18,500
Annual depreciation using straight-line (5-year life): $ 41,000

Annual income tax cash savings from difference in depreciation deduction:


($41,000 – $18,500)  0.25 = $5,625 (in favor of new machine)

1d.
Old New
Machi Machi
ne ne
Original cost $180,000 $225,000

Total depreciation 166,500 205,000


Book value of machines on Dec. 31, 2021 13,500 20,000
Terminal disposal price of machines on Dec. 31, 2021 10,000 18,000
Loss on disposal of machines 3,500 2,000
Add tax savings on loss (25% of $3,500; 25% of $2,000) 875 500
After-tax cash flow from terminal disposal of
machines ($10,000 + $875; $18,500 + $500) $ 10,875 $ 18,500

Difference in after-tax cash flow from terminal disposal of machines:


$18,500 – $10,875 = $7,625 (in favor of new machine)
2. The Kuhl Brothers should retain the old equipment because the net present value of the
incremental cash flows from the new machine is negative. The computations, using the results of
requirement 1, are presented below. In this format, the present value factors appear at the bottom.
All cash flows, year by year, are then converted into present values.

21-51
After-Tax Cash Flows
2016a 2017 2018 2019 2020 2021
Initial machine investment $(225,000)
Current disposal price of old machine 75,000
Tax savings from loss on disposal of
old machine 7,750
Recurring after-tax cash-operating savings
Variable $18,000 $18,000 $18,000 $18,000 $18,0
Fixed 3,000 3,000 3,000 3,000 3,0
Income tax cash savings from difference
in
depreciation deductions 5,625 5,625 5,625 5,625 5,6
Additional after-tax cash flow from
terminal disposal of new machine
over old machine _________ _______ _______ _______ _______ _ 7,6
$(1
42,
250
Net after-tax cash flows ) $26,625 $26,625 $26,625 $26,625 $34,2
_
1.
00
Present value discount factors (at 8%) 0 0.926 0.857 0.794 0.735 0.681
$(1
42,
250
Present value ) $24,655 $22,818 $21,140 $19,569 $23,3
$
(30
,74
Net present value 4)
a
More precisely, January 1, 2017

3. Let $X be the additional recurring after-tax cash operating savings required each year to
make NPV = $0.
The present value of an annuity of $1 per year for 5 years discounted at 8% = 3.993.

To make NPV = 0, Kuhl Brothers needs to generate cash savings with NPV of $30,744.
That is $X × (3.993) = $30,744
X = $30,744 ÷ 3.993
= $7,699.47

The Kuhl Brothers must generate additional annual after-tax cash operating savings of
$7,699.47.

21-40 Recognizing cash flows for capital investment projects. Johnny Buster owns
Entertainment World, a place that combines fast food, innovative beverages, and arcade

21-52
games. Worried about the shifting tastes of younger audiences, Johnny contemplates
bringing in new simulators and virtual reality games to maintain customer interest.
As part of this overhaul, Johnny is also looking at replacing his old Guitar Hero equipment
with a Rock Band Pro machine. The Guitar Hero setup was purchased for $25,200 and has
accumulated depreciation of $23,000, with a current trade-in value of $2,700. It currently costs
Johnny $600 per month in utilities and another $5,000 a year in maintenance to run the Guitar
Hero equipment. Johnny feels that the equipment could be kept in service for another 11 years,
after which it would have no salvage value.
The Rock Band Pro machine is more energy efficient and durable. It would reduce the
utilities costs by 30% and cut the maintenance cost in half. The Rock Band Pro costs $49,000
and has an expected disposal value of $5,000 at the end of its useful life of 11 years.
Johnny charges an entrance fee of $5 per hour for customers to play an unlimited number of
games. He does not believe that replacing Guitar Hero with Rock Band Pro will have an impact
on this charge or materially change the number of customers who will visit Entertainment World.

Required:
1. Johnny wants to evaluate the Rock Band Pro purchase using capital budgeting techniques. To
help him, read through the problem and separate the cash flows into four groups: (1) net
initial investment cash flows, (2) cash flow savings from operations, (3) cash flows from
terminal disposal of investment, and (4) cash flows not relevant to the capital budgeting
problem.
2. Assuming a tax rate of 40%, a required rate of return of 8%, and straight-line depreciation
over the remaining useful life of equipment, should Johnny purchase Rock Band Pro?

SOLUTION

(35 min.) Recognizing cash flows for capital investment projects.

1. Partitioning relevant cash flows into categories:

(1) Net initial investment cash flows:


- The $49,000 cost of the new Rock Band Pro

- The disposal value of Guitar Hero, $2,700, is a cash inflow

- The book value of Guitar Hero $2,200 ($25,200 − $23,000), relative to the disposal
value of $2,700, yields a taxable gain of $500 ($2,700 − $2,200) that leads to a cash
outflow for taxes of $500  Tax Rate

(2) Cash flow savings from operations:


- The 30% savings in utilities cost per year of $2,160 (30% × $600 per month × 12
months) results in cash inflow from operations after tax of $2,160  (1 − Tax Rate)

- The savings of half the maintenance costs per year of $2,500 (50% × $5,000) results in
a cash inflow from operations after tax of $2,500  (1 − Tax Rate)

21-53
- Annual depreciation of ($49,000 − $5,000) ÷ 11 years = $4,000 on Rock Band Pro,
relative to the ($2,200 − $0) ÷ 11 years = $200 depreciation on current Guitar Hero leads
to additional tax savings of $3,800 × Tax Rate

(3) Cash flows from terminal disposal of investment:


- The $5,000 salvage value of Rock Band Pro minus the $0 salvage value of the old
Guitar Hero equipment is a terminal cash flow at the end of Year 11. There are no tax
effects because both systems are planned to be disposed of at book value.

(4) Data not relevant to the capital budgeting decision:


- The $5 per hour charge for customers, since it would not change whether or not Johnny
got the new machine

- The $25,200 original cost of the Guitar Hero setup

2. Net present value of the investment:

Net initial investment


Initial investment in Rock Band Pro $(49,000)
Current disposal value of Guitar Hero 2,700
Tax on gain on sale of Guitar Hero, 40% × $500 (200)
Net initial investment $(46,500)

Annual after-tax cash flow from operations (excl. deprn. effects)


After-tax savings in utilities costs, $2,160 × (1−0.40) $ 1,296
After-tax savings in maintenance costs, $2,500 × (1−0.40) 1,500
Annual after-tax cash flow from operations $ 2,796
Income-tax cash savings from annual additional depreciation
deductions ($4,000 − $200) × 40% $ 1,520

After-tax cash flow from terminal disposal of machines $ 5,000

These four amounts can be combined to determine the NPV at an 8% discount rate.

Present value of net initial investment, $(46,500) × 1.000 $(46,500)


Present value of 11-year annuity of annual after-tax cash flow
from operations (excl. deprcn. effects), $2,796 × 7.139 19,961
Present value of 11-year annuity of income-tax cash savings from
annual depreciation deductions, $1,520 × 7.139 10,851
Present value of after-tax cash flow from terminal disposal of
machines, $5,000 × 0.429 2,145
Net present value $(13,543)

At the required rate of return of 8%, the net present value of the investment in the Rock Band Pro
machine is substantially negative. Johnny should therefore not make the investment.

21-54
21-41 NPV, inflation and taxes. Fancy Foods is considering replacing all 12 of its meat scales
with new, digital ones. The old scales are fully depreciated and have no disposal value. The new
scales cost $120,000 (in total). Because the new scales are more efficient and more accurate than
the old scales, Fancy Foods will have annual incremental cash savings from using the new scales
in the amount of $30,000 per year. The scales have a 6-year useful life and no terminal disposal
value and are depreciated using the straight-line method. Fancy Foods requires a 6% real rate of
return.

Required:
1. Given the preceding information, what is the net present value of the new scales? Ignore
taxes.
2. Assume the $30,000 cost savings are in current real dollars and the inflation rate is 4%.
Recalculate the NPV of the project.
3. Based on your answers to requirements 1 and 2, should Fancy Foods buy the new meat
scales?
4. Now assume that the company’s tax rate is 25%. Calculate the NPV of the project
assuming no inflation.
5. Again assuming that the company faces a 25% tax rate, calculate the NPV of the project
under an inflation rate of 4%.
6. Based on your answers to requirements 4 and 5, should Fancy Foods buy the new meat
scales?

SOLUTION

(25 min.) NPV, inflation and taxes.

1. Without inflation or taxes, this is a simple net present value problem using an 8%
discount rate

Present value of initial investment, $(120,000) × 1.000 $(120,000)


Present value of 6-year annuity of annual cash savings:
$30,000 × 4.917 14
7,5
10
Net present value $
27,
51
0

2. With inflation, we adjust each year’s cash flow for the inflation rate to get nominal cash
flows and then discount each cash flow separately using the nominal discount rate.

Nominal rate = (1 + real rate) × (1 + inflation rate) − 1


Nominal rate = (1.06) × (1.04) − 1 = 1.1024 – 1 = 0.1024 or 10% (approx.)

Cash Flow Cumulative Cash Inflows Present Value

21-55
Period (Real Dollars) Inflation Rate (Nominal Dollars) Factor, 10% Present Value
(1) (2) (3) = (1) × (2) (4) (5) = (3) × (4)
1 $30,000 1.040 $31,200 0.909 $
28,36
1
2 $30,000 1.0821 $32,460 0.826 26,81
2
3 $30,000 1.125 $33,750 0.751 25,34
6
4 $30,000 1.170 $35,100 0.683 23,97
3
5 $30,000 1.217 $36,510 0.621 22,67
3
6 $30,000 1.265 $37,950 0.564 21,
404
Total present value of annual net cash inflows in nominal dollars 148,5
69
Present value of initial investment, $(120,000) × 1.000 (120,000)
Net present value $
28,56
9
1
1.082 = (1.04)2

3. Both the unadjusted and adjusted NPV are positive. Based on financial considerations
alone, Fancy Foods should buy the new meat scales. However, the effect of taxes should also be
considered, as well as any pertinent nonfinancial issues, such as faster customer service time and
increased confidence by customers.

4. Initial equipment investment $(120,000)

Annual cash flow from operations (excl. deprn. effects) $30,000


Deduct income tax payments (0.25 × $30,000) 7,500
$
22
,5
Annual after-tax cash flow from operations (excl. deprn. effects) 00

$
5,
Income tax cash savings from annual depreciation deductions 00
(0.25 × $20,000)1 0
1
Depreciation deductions = ($120,000 – $0) / 6 = $20,000

The terminal disposal price of the equipment is equal to the book value at disposal = $0, so the
above three amounts suffice to determine the NPV at a 6% discount rate.

21-56
Present value of net initial investment, $(120,000) × 1.000 $(120,000)
Present value of 6-year annuity annual after-tax cash flow from operations,
$22,500 × 4.917 110,633
Present value of 6-year annuity of income tax cash savings from
annual depreciation deductions, $5,000 × 4.917 24,585
Net present value $ 15,218

5. As in the previous section, with inflation, we adjust each year’s cash flow for the
inflation rate to get nominal cash flows and then discount each cash flow separately using the
nominal discount rate.

Nominal rate = (1 + real rate) × (1 + inflation rate) −1


Nominal rate = (1.06)(1.04) −1 = 1.1024 – 1 = .1024 or 10% (approx.)

After Tax
Cash Flow Cumulative Cash Inflows Cash Present Value
Period (Real Dollars) Inflation Rate (Nominal Dollars) Flows Factor, 10% Present Value
(1) (2) (3) = (1) × (2) (4) = 0.75 × (3) (5) (6) = (4) × (5)
1 $ 30,000 1.040 $31,200 $ 23,400 0.909 $ 21,271
2 30,000 1.082 32,460 24,345 0.826 20,109
3 30,000 1.125 33,750 25,313 0.751 19,010
4 30,000 1.170 35,100 26,325 0.683 17,980
5 30,000 1.217 36,510 27,383 0.621 17,005
6 30,000 1.265 37,950 28,463 0.564 16,053
Total present value of annual net cash inflows (excl. depreciation. effects)
$11
1,42
8
Present value of 6-year annuity of income-tax cash savings from
annual depreciation deductions, $5,000 × 4.917 2
4,58
5
Present value of initial investment $(120,000) × 1.000 (1
20,0
00)
$
16,0
Net present value 13

6. Without inflation, we obtain a positive NPV; and, with inflation NPV is still positive, so
Fancy Foods is better off purchasing the new meat scales. NPV is obtained with an inflation
estimate of 4%. If a careful review of this forecasted inflation rate results in a higher rate of
inflation, Fancy Foods should recalculate the NPV to determine whether the purchase of the
registers is still in its best interest. With a higher inflation rate, a higher tax rate, or a higher
required rate of return, the NPV could be negative and then their decision would change and they
would not purchase the new meat scales.

21-57
21-42 NPV of information system, income taxes. Saina Supplies leases and sells materials,
tools, and equipment and also provides add-on services such as ground maintenance and
waterproofing to construction and mining sites. The company has grown rapidly over the past
few years. The owner, Saina Torrance, feels that for the company to continue to scale, it needs to
install a professional information system rather than relying on intuition and Excel analyses.
After some research, Saina’s CFO reports back with the following data about a data warehousing
and analytics system that she views as promising:

■ The system will cost $750,000. For tax purposes, it can be depreciated straight-line to a zero
terminal value over a 5-year useful life. However, the CFO expects that the system will still
be worth $50,000 at that time.
■ There is an additional $75,000 annual fee for software upgrades and technical support from
the vendor.
■ The ability to provide better services and to target and reach more clients as a result of the
new system will directly result in a $500,000 increase in revenues for Saina in the first year
after installation. Revenues will grow by 5% each year thereafter. Saina’s contribution
margin is 60%.
■ Due to greater efficiency in ordering and dispatching supplies, as well as in collecting
receivables, the firm’s working-capital requirements will decrease by $100,000.
■ Saina will also be able to reduce the amount of warehouse space it currently leases, saving
$40,000 annually in the process.
■ Saina Supplies pays an income tax of 30% and requires an after-tax rate of return of 12%.

Required:
Assume that all cash flows occur at year-end except for initial investment amounts.

1. If Saina decides to purchase and install the new information system, what is the expected
incremental after-tax cash flow from operations during each of the 5 years?
2. Compute the net present value of installing the information system at Saina Supplies.
3. In addition to the analysis in requirement 2, what nonfinancial factors you would consider in
making the decision about the information system?

SOLUTION

(60 min.) NPV of information system, income taxes.

1. Initial investment (Year 0): $750000

Working-capital investment:
Reduced working capital of $100000 at end of Year 0.
Increased working capital of $100000 at end of Year 5.

Depreciation on initial investment: $750000  5 years = $150000 per year

Income tax cash savings from annual depreciation deductions: $150000 × 0.30 = $45000

21-58
After-tax cash flow from disposal of JIT system at end of Year
5: $50000 × (1– 0.30) = $35000

Annual after-tax cash flow from operations:

  Year 1 Year 2 Year 3 Year 4 Year 5


Incremental revenues
(5% annual growth) $500,000 $525,000 $551,250 $578,813 $607,753
Incremental contribution margin
(60%  incremental revenues) $300,000 $315,000 $330,750 $347,288 $364,652
Rent savings 40,000 40,000 40,000 40,000 40,000
Deduct increase in software
upgrades and tech support costs (75,000) (75,000) (75,000) (75,000) (75,000)
Annual pre-tax incremental
cash inflow from operations 265,000 280,000 295,750 312,288 329,652
Deduct income tax payments
(30%) 79,500 84,000 88,725 93,686 98,896
Annual after-tax incremental
cash inflow from operations $185,500 $196,000 $207,025 $218,602 $230,756

2. Solution Exhibit 21-43 reports the net present value to be $214,506.

3. Saina will have a NPV of $214,506 with the new data warehousing and analytics system.
Based on financial quantitative factors, this is an attractive investment. Qualitative factors could
make the system even more attractive. For example, if a competitor adopts the new information
system but Saina does not, Saina could be at a sizable competitive disadvantage. Not adopting
the information system does not mean the status quo will remain. Saina’s workers can also gain
additional expertise when using the data warehousing and analytics system that can be
beneficially employed on other projects.

21-59
SOLUTION EXHIBIT 21-43
Present
Value Sketch of Relevant After-Tax Cash Flows
Total Discount
Present Factors
Value at 12% Year 0 Year 1 Year 2 Year 3 Year 4 Year 5

1a. Net initial


investment $(750000) 1.000 $(750000)
1b. Working
capital decrease 100,000 1.000 $100000
2a. Annual after-
tax cash flow
from operations
Year 1 165,652 0.893 $185,500
Year 2 156,212 0.797 $196000
Year 3 147,402 0.712 $207,025
Year 4 139,031 0.636 $218,602
Year 5 130,839 0.567 $230,756
2b. Income tax cash
savings from annual
deprecn. Deductions
Year 1 40,185 0.893 $45000
Year 2 35,865 0.797 $45000
Year 3 32,040 0.712 $45000
Year 4 28,620 0.636 $45000
Year 5 25,515 0.567 $45000
3. After-tax cash flow from:
a. Terminal
disposal of
machine 19,845 0.567 $35000
b. Increase in
working capital (56,700) 0.567 $(100000)
Net
present value $214,506

21-60
Try It! 21-1

a. From Appendix A, Table 4, the present value of a $1 annuity for 8 years at 8% is 5.747

Net present value = $65,000 (5.747) – $250,000


= $373,555 – $250,000
= $123,555

b. $250,000 = Present value of annuity of $65,000 at R% for 8 years, or what factor (F) in the
table of present values of an annuity will satisfy the following equation.

$250,000 = $65,000F
F = 250,000/65,000
= 3.85

On the 8-year line in the table for the present value of annuities (Appendix A, Table 4), the
column closest to 3.85 indicates that it is between a rate of return of 18% and 20%.

We can then interpolate between these two rates of return:

Present Value Factors


18% 4.078 4.078
IRR rate – 3.850
20% 3.837 ––
Difference 0.241 0.228

Internal rate of return = 18% + (.228/.241) * (2%)


= 18% + (.946) * (2%)
= 19.89%

Try It! 21-2

a. Payback period = $250,000 ÷ $65,000


= 3.85 years.

b. Discounted payback period:

Period Cash Savings Discount Discounted Cumulative Unrecovered


Factor (8%) Cash Savings Discounted Investment
Cash Savings
0 ($250,000)
1 $65,000 .926 $60,190 $ 60,190 ($189,810)
2 $65,000 .857 $55,705 $115,895 ($134,105)
3 $65,000 .794 $51,610 $167,505 ($ 82,495)
4 $65,000 .735 $47,775 $215,280 ($ 34,720)
5 $65,000 .681 $44,265 $259,545

21-61
It is evident that the discounted payback happens between years 4 and 5.
To get an approximate sense for when, note that $34,720/$44,625 = 0.78
So, the discounted payback period = 4.78 years

Try It! 21-3

a. Accrual accounting rate of return based on net initial investment:

Net initial investment = $250,000


Estimated useful life = 8 years
Annual straight-line depreciation = $250,000 ÷ 8 = $31,250

Accrual accounting Increase in expected average annual operating income


rate of return = Net initial investment

= ($65,000 – $31,250) / $250,000


= $33,750 / $250,000
= 13.5%

b. Accrual accounting rate of return based on average investment:

Average investment = ($250,000 + $0)/2 = $125,000

Accrual accounting Increase in expected average annual operating income


rate of return = Average investment

= ($65,000 – $31,250) / $125,000


= $33,750 / $125,000
= 27.0%

c. Other than the quantitative metrics used to evaluate the new computer system, qualitative
factors that Home Value should consider are:
 Issues related to the financing of the project, and the availability of capital to pay for
the system.
 The effect of the system on employee morale, particularly those displaced by the
system. Salesperson expertise and real-time help from experienced employees is key
to the success of a home improvement store.
 The benefits of the new system for customers (faster checkout, fewer errors).
 The upheaval of installing a new computer system. Its useful life is estimated to be 8
years. This means that Home Value could face this upheaval again in 8 years. Also,
ensure that the costs of training and other “hidden” start-up costs are included in the
estimated $250,000 cost of the new computer system.

21-62
Try It! 21-4

Present value of keeping current system:

Predicted PV Factor PV of Cash


Cash Flows Year(s) @ 14% Flows
Overhaul $(30,250) 0 1.000 $(30,250)
Annual operation costs (69,300) 1-5 3.433 (237,907)
Salvage value at end 8,800 5 0.519 4,567
Net present value $(263,590)

Present value of new system:

Predicted PV Factor PV of Cash


Cash Flows Year(s) @ 14% Flows
Investment ($162,800) 0 1.000 ($162,800)
Salvage value, old 44,000 0 1.000 $44,000
Annual operation costs (52,800) 1-5 3.433 ($181,262)
Salvage value at end 38,500 5 0.519 $19,982
Net present value ($280,080)

Overhauling the existing system is the better option by $(263,590) - $(280,080), or $16,490.

21-63

Вам также может понравиться